This discussion board is a place to discuss our Daily Challenges and the math and science
related to those challenges. Explanations are more than just a solution — they should
explain the steps and thinking strategies that you used to obtain the solution. Comments
should further the discussion of math and science.
When posting on Brilliant:
Use the emojis to react to an explanation, whether you're congratulating a job well done , or just really confused .
Ask specific questions about the challenge or the steps in somebody's explanation. Well-posed questions can add a lot to the discussion, but posting "I don't understand!" doesn't help anyone.
Try to contribute something new to the discussion, whether it is an extension, generalization or other idea related to the challenge.
Stay on topic — we're all here to learn more about math and science, not to hear about your favorite get-rich-quick scheme or current world events.
Markdown
Appears as
*italics* or _italics_
italics
**bold** or __bold__
bold
- bulleted - list
bulleted
list
1. numbered 2. list
numbered
list
Note: you must add a full line of space before and after lists for them to show up correctly
# I indented these lines
# 4 spaces, and now they show
# up as a code block.
print "hello world"
# I indented these lines
# 4 spaces, and now they show
# up as a code block.
print "hello world"
Math
Appears as
Remember to wrap math in \( ... \) or \[ ... \] to ensure proper formatting.
2 \times 3
2×3
2^{34}
234
a_{i-1}
ai−1
\frac{2}{3}
32
\sqrt{2}
2
\sum_{i=1}^3
∑i=13
\sin \theta
sinθ
\boxed{123}
123
Comments
While Muirhead works, another viable option is a generalization of rearrangement by taking four copies of {x,y,z}. (WLOG x≥y≥z≥0) Since {x,y,z} is monotonic decreasing, we have
Muirhead states that for any two sets of numbers a1,a2,…an and b1,b2,…bn, if (a1,a2,…an)≻(b1,b2,…bn), then sym∑x1a1x2a2⋯xnan≥sym∑x1b1x2b2⋯xnbn
Search up Majorization if you do not know what ≻ (which means majorizes) means.
But Muirhead is basically just a generalized AM-GM. Setting the first set of numbers as 1,0,…0 and the second set as n1,n1,…n1, we see that (1,0,…0)≻(n1,n1,…n1) and using Muirhead's gives us the familiar n-term AM-GM.
@Xuming Liang
–
It's good to finally see some inequality actions going on here, I've never been interested in inequalities until recently :) Here's one I feel kinda proud for solving it:
a,b,c are positive reals, ab+bc+ca=1 prove 3a1+6b+3b1+6c+3c1+6a≤abc1.
P.S. Good think on AoPS there's a quick way for you to copy and paste latex..
Since anything provable using Muirhead (which was the instakill solution that I suggested) could be proven using AM-GM, the challenge was to prove my suggested inequalities using AM-GM.
Easy Math Editor
This discussion board is a place to discuss our Daily Challenges and the math and science related to those challenges. Explanations are more than just a solution — they should explain the steps and thinking strategies that you used to obtain the solution. Comments should further the discussion of math and science.
When posting on Brilliant:
*italics*
or_italics_
**bold**
or__bold__
paragraph 1
paragraph 2
[example link](https://brilliant.org)
> This is a quote
\(
...\)
or\[
...\]
to ensure proper formatting.2 \times 3
2^{34}
a_{i-1}
\frac{2}{3}
\sqrt{2}
\sum_{i=1}^3
\sin \theta
\boxed{123}
Comments
While Muirhead works, another viable option is a generalization of rearrangement by taking four copies of {x,y,z}. (WLOG x≥y≥z≥0) Since {x,y,z} is monotonic decreasing, we have
x⋅x⋅x⋅x+y⋅y⋅y⋅y+z⋅z⋅z⋅z ≥x⋅x⋅y⋅z+y⋅y⋅z⋅x+z⋅z⋅x⋅y
A solution by Muirhead that I discovered while writing the problem is as follows:
Note that (2,0,0)≻(1,21,21).
Thus, by Muirhead, 2(x2+y2+z2)≥2(xyz+yzx+zxy) and the result follows.
The challenge I give you is to try to prove it using AM-GM. It shouldn't be too difficult.
Log in to reply
By AM-GM,
4x2+x2+y2+z2≥4x4y2z2
so 21x2+41y2+41z2≥xyz. Similarly, if we duplicate the y2 and z2 terms, we get
41x2+21y2+41z2≥yzx, and
41x2+41y2+21z2≥zxy. Adding these inequalities together, we have
x2+y2+z2≥xyz+yzx+zxy as requested. □
Log in to reply
Ah, you beat me to it with the same solution I posted. Kudos to you on being first.
By AM-GM, 42x2+y2+z2=4x2+x2+y2+z2≥4x4y2z2.
Similarly, 4x2+2y2+z2≥4x2y4z2 and 4x2+y2+2z2≥4x2y2z4. Note that 4x4y2z2=x2yz=xyz, which can also apply to the other three radicals.
Dividing through, we have this system of inequalities.
21x2+41y2+41z241x2+21y2+41z241x2+41y2+21z2≥xyz≥yzx≥zxy
Adding these together yields x2+y2+z2≥xyz+yzx+zxy, as desired.
Q.E.D.
@Daniel Liu, Can you please explain your Muirhead solution a little further? I'm not familiar with that.
Log in to reply
Muirhead states that for any two sets of numbers a1,a2,…an and b1,b2,…bn, if (a1,a2,…an)≻(b1,b2,…bn), then sym∑x1a1x2a2⋯xnan≥sym∑x1b1x2b2⋯xnbn
Search up Majorization if you do not know what ≻ (which means majorizes) means.
But Muirhead is basically just a generalized AM-GM. Setting the first set of numbers as 1,0,…0 and the second set as n1,n1,…n1, we see that (1,0,…0)≻(n1,n1,…n1) and using Muirhead's gives us the familiar n-term AM-GM.
We could apply AM-GM on yz≤2y+z which would lead to proving the well known x2+y2+z2≥xy+yz+xz
Log in to reply
It is also true that xy+yz+zx≥xyz+yzx+zxy
Can you prove that?
Log in to reply
Well by doing what I said this is actually what you get :).....: xyz+yzx+zxy≤x2y+z+y2x+z+z2y+x=xy+yz+zx
Log in to reply
However, it is also true that (xyz+yzx+zxy)3≥27x2y2z2
How about that one?
Log in to reply
Log in to reply
a,b,c are positive reals, ab+bc+ca=1 prove 3a1+6b+3b1+6c+3c1+6a≤abc1.
P.S. Good think on AoPS there's a quick way for you to copy and paste latex..
How about this one?
2xyz+2yzx+2zxy≥sym∑3xy2z3
@Michael Tang @Trevor B. Try your hand at this one.
Log in to reply
1,21,21 majorizes 1,32,31, use Muirhead to obtain the result.
Log in to reply
@Bogdan Simeonov
Sorry, but the point was to prove it using AM-GMSince anything provable using Muirhead (which was the instakill solution that I suggested) could be proven using AM-GM, the challenge was to prove my suggested inequalities using AM-GM.
Log in to reply